Difference between revisions of "2021 AMC 12B Problems/Problem 6"

m (Solution)
Line 4: Line 4:
 
<math>\textbf{(A)} ~1.5 \qquad\textbf{(B)} ~3 \qquad\textbf{(C)} ~4 \qquad\textbf{(D)} ~4.5 \qquad\textbf{(E)} ~6</math>
 
<math>\textbf{(A)} ~1.5 \qquad\textbf{(B)} ~3 \qquad\textbf{(C)} ~4 \qquad\textbf{(D)} ~4.5 \qquad\textbf{(E)} ~6</math>
 
==Solution==
 
==Solution==
<math>\boxed{\textbf{(A)} ~1.5}</math> Dear MAA, Please stop filling cones with water
+
<math>\boxed{\textbf{(A)} ~1.5}</math> Dear MAA, Please stop filling cones with water and then dumping them into cylinders

Revision as of 18:46, 11 February 2021

Problem

An inverted cone with base radius $12\mathrm{cm}$ and height $18\mathrm{cm}$ is full of water. The water is poured into a tall cylinder whose horizontal base has radius of $24\mathrm{cm}$. What is the height in centimeters of the water in the cylinder?

$\textbf{(A)} ~1.5 \qquad\textbf{(B)} ~3 \qquad\textbf{(C)} ~4 \qquad\textbf{(D)} ~4.5 \qquad\textbf{(E)} ~6$

Solution

$\boxed{\textbf{(A)} ~1.5}$ Dear MAA, Please stop filling cones with water and then dumping them into cylinders